If you're seeing this message, it means we're having trouble loading external resources on our website.

If you're behind a web filter, please make sure that the domains *.kastatic.org and *.kasandbox.org are unblocked.

Main content

Course: LSAT > Unit 1

Lesson 5: Analytical Reasoning – Worked examples

Ordering setup | Completely determines | Worked example

Watch a demonstration of one way to approach a "New info: must be true" question on an ordering setup from the analytical reasoning section of the LSAT.

Want to join the conversation?

  • blobby green style avatar for user Nico R
    At how can M or S be the element in slot 601? Per the 3rd rule, M must be built earlier than 604. If S were in slot 601 then the earliest element M could be is on 604, which violates the third rule. So elements G and element M must be in slot 601. The reason answer choice A is wrong is because for slot 604 and slot 605 element H or element S could fill in either.

    601-G,M
    602-L
    603-F
    604-H/S
    605-H/S

    The ambiguity with slots 604 and 605 is why answer choice A is incorrect.
    (25 votes)
    Default Khan Academy avatar avatar for user
  • blobby green style avatar for user nyplacnew
    I think this question can be quickly solved by realizing S is the free-lancer of the team who is not restricted by any rules. So the added rule restricting S would most likely restrict everything
    (13 votes)
    Default Khan Academy avatar avatar for user
  • blobby green style avatar for user Anicham
    why can't the answer be C ?
    (1 vote)
    Default Khan Academy avatar avatar for user

Video transcript

- [Instructor] Before you dive into the questions for this setup make sure to watch the setup video in which we've created our initial diagram using the rules and in which we made deductions from those rules. This question asks the years in which each of the monuments were begun can be completely determined if which one of the following is discovered to be true? So this question is essentially asking us for the choice that provides information that would allow us to know where every monument goes. Essentially our diagram would be filled out completely and usually these question types are best tackled by diving right in with our pencils to test each choice. The wrong choices won't let us get to that finish line so as soon as a choice leads to a dead end we can move on to testing the next one. Let's try A. F was begun in 603. In this case we do know that G is in 601 and L is in 602 because G is before L and L is before F but we don't know which of the other monument is in 601 with G. It could be M or S so we can move on. B, what if G was begun in 602? Well we would know that M and S are the monuments in 601 and then that L has to be in 603 since it can't be four or five but we don't know what order F and H are in so almost, not quite. C, H was begun in 605. Well we can't deduce anything for certain from here. Either F or S could be in 604 for example so there's just too many question marks here. D, M was begun in 602. We do know that GS are in 601 if M is in 602 and then we can deduce that L is in 603 but we don't know what order F and H are in just like with choice B. That leaves choice E which says that S was begun in 604. Well if S is in 604 then H has to be in 605. That would leave the GLF chain to be one, two and three and then only M is left to be with G in one so E allows us to place each of the elements. Don't be afraid to use your pencil to your advantage on test day because it's gonna be faster one question types such as these in which it would be pretty difficult to work it all out in your head so the answer here is E.